Marshall wants to buy a picture frame. The original price is $15,
How much will Marshall pay if he buys it during the sale?
SALE
25% off
original price

Answers

Answer 1
15x.25=3.75
15-3.75=11.25.

The answer is $11.25

Related Questions

the length of a rectangly is eight more than twice its width. if the length is less than 34, then what is the width?

Answers

Answer:

13

Step-by-step explanation:

I think u mean if the length is 34. If so heres how to work it out.

l = 2w + 8

l = 34.

34 - 8 = 26

26 / 2 = 13.

Width = 13

Answer:

x < 13

Step-by-step explanation:

Write an equation for the width

2x

→ Write an equation for the length

8 + 2x

→ Set up an equation

8 + 2x < 34

→ Solve

x < 13

One leg of a 45°-45°-90° triangle is 9. Find the lengths of the other sides.

Answers

Answer:

  9, 9√2

Step-by-step explanation:

The ratios of side lengths in a 45°-45°-90° triangle are ...

  1 : 1 : √2

The other leg is the same as the first, and the hypotenuse is √2 times that.

The other sides are 9 and 9√2.

Josie owns 21 books. She buys 6 books every week. Write the recursive sequence to describe how number books she has.

Answers

Answer:

a1=21

an=an-1+6

Step-by-step explanation:

please help ASAP! i have been struggling on this question and need help!

Answers

Answer:

841.53

Step-by-step explanation:

equation for annual intrest is

[tex]I*y^x=f[/tex]

where I is your initial amount, y is equal to [tex]1+(interest /100)[/tex]

and x is equal to the amount of years you're looking for, with f being the final balance

so for this questions, the equation is as follows

[tex]600*(1+7/100)^5=f\\600*1.07^5=f\\600*1.40255=f\\841.53=f\\[/tex]

If line segment AB represents 25​%, what is the length of a line segment that is​ 100%?

Answers

Answer:32 Inches
I Hope This Helps

Two same-sized triangular prisms are attached to a rectangular prism as shown below.
If a = 20 cm, b = 13 cm, c = 12 cm, d = 5 cm, and e = 8 cm, what is the surface area of the figure?
a 1,208 square centimeters
b 1,592 square centimeters
c 1,004 square centimeters
d 1,400 square centimeters

Answers

Answer:

answer will be a 1,208 square centimeters

help me with this math problem please​

Answers

Answer:

BC ≅ EF

Step-by-step explanation:

Since the triangles are congruent then corresponding sides are congruent

ABC ≅ DEF

The corresponding sides are BC and EF

Help help math math

Answers

Answer:

5/1

Step-by-step explanation:

the y is 5units and the x is 1unit

Answer:

7/4

Step-by-step explanation:

Use rise over run to solve

4-(-3)         7

-------- =      -

6-2            4

A restaurant tablecloth is 98 inches across. What is the approximate circumference?

Answers

Answer:

The circumference of a circle with diameter of 98 in. is about 308 in.

Step-by-step explanation:

We know the diameter. Using the diameter, we can find the circumference.

=> Diameter = 98 in.=> Radius = 98/2=> Radius = 49 in.

Circumference = 2πr

=> 2 x 22/7 x 49=> 2 x 22 x 7=> 308 in.Conclusion:

Therefore, the circumference of a circle with diameter of 98 in. is about 308 in.

Hoped this helped.

[tex]BrainiacUser1357[/tex]

2.03 of 0.045 is what number?

Answers

Answer:

2.03% of 0.045=0.0009135

Step-by-step explanation:

To get the solution, we are looking for, we need to point out what we know.

1. We assume, that the number 0.045 is 100% - because it's the output value of the task.

2. We assume, that x is the value we are looking for.

3. If 0.045 is 100%, so we can write it down as 0.045=100%.

4. We know, that x is 2.03% of the output value, so we can write it down as x=2.03%.

5. Now we have two simple equations:

1) 0.045=100%

2) x=2.03%

where left sides of both of them have the same units, and both right sides have the same units, so we can do something like that:

0.045/x=100%/2.03%

6. Now we just have to solve the simple equation, and we will get the solution we are looking for.

7. Solution for what is 2.03% of 0.045

0.045/x=100/2.03

(0.045/x)*x=(100/2.03)*x       - we multiply both sides of the equation by x

0.045=49.261083743842*x       - we divide both sides of the equation by (49.261083743842) to get x

0.045/49.261083743842=x

0.0009135=x

x=0.0009135

Answer:

0.09135

Step-by-step explanation:

when it says "of" a number, you simply multiply them

Leah goes to Buffalo Wild Wings for lunch. Her total bill was $35.50. She wants
to leave a tip that is 18% of the total bill. How much should she leave for the
server? How much did she spend in all?

Answers

Answer:

$41.89

Step-by-step explanation:

18% of %35.50 is $6.39.

$6.39+$35.50= $41.89

Answer:

She should leave $6.39 tip for the server. She paid $41.89 in all. Hope this helped! if the answer is wrong im super sorry!

Step-by-step explanation:

0.18 x 35.50 = 6.39

6.39 + 30.50 = 41.89

A rock is 7 feet below the surface of a river. If its position can be recorded as −7 feet, what would the position of 0 represent? (5 points)

Question 3 options:

1)

On the surface of the river


2)

At the bottom of the river


3)

7 feet above the surface of the river


4)

7 feet below the surface of the river

Answers

Answer:

A on the surface of the river

Step-by-step explanation:

100 persent correct

9.
10. Find two numbers such that five times the larger number plus twice the smaller is 70, and four
times the larger minus twice the smaller is 38.

Answers

The two numbers are 12 and 5.

Two equations can be derived from this question

5x + 2y = 70 equation 1

4x - 2y = 38 equation 2

Where:

x = larger number

y = smaller number

In order to determine the  larger number, the following steps hare to be taken:

Add equation 2 and 1

9x = 108

Divide both sides by 9

x = 108/9

x = 12

In order to determine the smaller number, substitute for x in equation 1

5(12) + 2y = 70

60 + 2y = 70

70 - 60 = 2y

10 = 2y

y = 5

To learn more about simultaneous equations, please check: https://brainly.com/question/25875552

If a trench is 102’-6” long by 5’-4” wide and 3’-3” deep, what is the volume of the soil that was removed per cubic feet?

Answers

Considering that the trench is modeled by a rectangular prism, the volume that was removed is 1776 cubic feet.

The volume of a rectangular prism of length l, width w and height h is given by:

[tex]V = lwh[/tex]

In this problem, we want the measures in feet, and considering that 1 feet is 12 inches:

The length is of 102' 6''(102 feet and 6 inches), hence [tex]l = 102.5[/tex].The width is of 5' 4'', hence [tex]w = 5.33[/tex].The depth(height) is of 3' 3'', hence [tex]h = 3.25[/tex].

Then:

[tex]V = lwh = 102.5(5.33)(3.25) = 1776[/tex]

The volume that was removed is 1776 cubic feet.

To learn more about the volume of rectangular prisms, you can take a look at https://brainly.com/question/17223528




Help help help math math math

Answers

Answer:

1/4

Step-by-step explanation:

the x decreases by 1 the y decreases by 4

Answer:

-4

Step-by-step explanation:

Quincy feeds his dog 2.75 cups of dog food each day. Each cup of dog food costs $1.25. He multiplies the numbers together to determine how much it costs to feed his dog each day. Which expression will result in the best estimate for Quincy to use to check his multiplication?
2 times 1
2 times 2
3 times 1
3 times 2

Answers

Answer:

3 times 1

Step-by-step explanation:

2.75 - 75 or in other words 7 is closer to 10 than 0, so well round it up to the next number, 3.

1.25 - 25 or in others words 2 is closer to 0 than 10, so we'll round it down to the nearest lowest whole number, 1.

our new expression: 3 × 1. hope that helps!

Approximately what portion of the beaker is filled?
A.3/4
B. 1/2
C. 5/9

Answers

i would say the answer is 3/4 :)

Answer:

The answer which I think is 3_4

PLS HELP ON GEOMETRY: select all the true statements

Answers

Answer:

E

Step-by-step explanation:

130 is what percent of 70?

Answers

Answer:

91%

Step-by-step explanation:

hopes this helps

Answer:

185.71%

Step-by-step explanation:

130 / 70 = 1.857142857

(1.857142857)(100) = 185.71%

(a) 95% of what is 57?

(b) What number is 35% of 40?

Answers

Answer:

a=54.15 and b=14

Step-by-step explanation:

-6x+6y=9 (1/2,2) please helppppp

Answers

Answer:

A. Solution

General Formulas and Concepts:

Pre-Algebra

Order of Operations: BPEMDAS

Brackets Parenthesis Exponents Multiplication Division Addition Subtraction Left to Right

Algebra I

Coordinate Planes

Coordinates (x, y)

Step-by-step explanation:

*Note:

A coordinate is a solution if both the left side and right side of the equation is equal.

Step 1: Define

Identify.

-6x + 6y = 9

(1/2, 2)

Step 2: Verify

Substitute in variables [Equation]:                                                                   -6(1/2) + 6(2) = 9[Order of Operations] Evaluate:                                                                      9 = 9

Since 9 does indeed equal 9, the coordinate (1/2, 2) is a solution.

B(14, 12)
A(-5,-2)
A. Find the midpoint of Segment AB

Answers

the midpoint formula is (x1+x2/2, y1+y2/2
so you would do (-5+14)/2=4.5. (-2+12)/2=5
(4.5, 5)

are 6/12 and 12/24 equavilent​

Answers

Answer:

yes because they're both half of each other

Answer:

Yes

Step-by-step explanation:

sixty students at gillette road middle school play a winter modified sport. if there are 500 students in the middle school, what percent of students play a sport

Answers

Answer:

60/500=0.12

so 12%

Hope This Helps!!!

Simplify the linear expression. −4x+5(−5x−1) Enter your answer in the box.

Answers

Answer:

-29x - 5

Step-by-step explanation:

-4x - 25x - 5 = -29x - 5

Answer:

- 29x - 5

Step-by-step explanation:

- 4x + 5(- 5x - 1) ← distribute the parenthesis by 5

= - 4x - 25x - 5 ← collect like terms

= - 29x - 5

Identify the digit in the ten thousandths place in every number.
1.) 12.00535
2.)54.01247
3.) 3.98056
4.) 0.00667
5.) 70.70071
6.) 0.07889
7.) 10.00898​

Answers

Answer:

Step-by-step explanation:

The decimal point will help you find your way here. Just to the right of the decimal is the tenths place. 0.4 is the number four tenths.

Next, farther to the right is the hundredths place-- 0.04 is the number four hundredths.

Next over is the thousandths place-- 0.004 is the number four thousandths. And finally, the next over is the ten thousandths place...like this, 0.0004 is the number four ten thousandths

So, 1) 12.00535 has a one in the tens place, a two in the ones place, zeros in the tenths and also the hundredths places, fives in the thousandths place and also the hundred thousandths place, AND, what you're looking for-- a 3 in the ten thousandths place.

1) 3

2) 4

3) 5

4) 6

5) 7

6) 8

7) 9

For ten thousandths place, you're looking at the fourth spot on the right side of the decimal.

Surface area of cone 5ft by 21ft

Answers

if the radius is 5 then the answer is 417.63

if the radius is 21 then the answer is 2809.61

Answer:

417.63

Step-by-step explanation:

Dev has a monthly food budget of $182. He maps the amount of money,x, he spends each month to the number of food items he buys. What are the constrains on the domain?

Answers

Using it's concept, it is found that the domain of the function is [tex]0 \leq x \leq 182[/tex], that is, the constraints are:

x is of at least $0.x is of at most $182.

The domain of a function is the set that contains all possible values for the functions.

In this problem, the input of the function is how much he spends on food.

His budget is of $182, hence, considering this and the fact that he cannot spend a negative amount on food, the constraints are that x is between $0 and his budget of $182(inclusive), and the domain is [tex]0 \leq x \leq 182[/tex].

To learn more about the domain of functions, you can take a look at https://brainly.com/question/25897115

The domain of a function is the set of input values the function can have.

The constraints on the domain are: the amount (x) is at least $0, and it is at most $182

From the question, Dev's budget on food is given as:

[tex]Budget = \$182[/tex]

This means that, he cannot spend more than $182.

Represent the amount with x.

So, we have:

[tex]x \le 182[/tex]

Also, he cannot spend a negative amount.

So, we have:

[tex]x \ge 0[/tex]

So, the domain is:

[tex]x \ge 0[/tex] and [tex]x \le 182[/tex]

Rewrite as:

[tex]0 \le x[/tex] and [tex]x \le 182[/tex]

Combine both inequalities

[tex]0 \le x \le 182[/tex]

Hence, the constraints on the domain are: the amount (x) is at least $0, and it is at most $182

Read more about domain at:

https://brainly.com/question/10197594

Please help?!! Hurry please!!

Answers

Answer:

C. [tex] \frac{ {m}^{2} - m - 2}{ {m}^{2} - 1} [/tex]

Explanation:

[tex] \frac{ {m}^{2} - m - 2}{ {m}^{2} - 1} \\ = \frac{ {m}^{2} - 2m + m - 2 }{ {(m)}^{2} - {(1)}^{2} } \\ = \frac{ m(m - 2) + 1(m - 2)}{(m - 1)(m + 1)} \\ = \frac{(m + 1)(m - 2)}{(m - 1)(m +1) } \\ = \frac{(m - 2)}{(m - 1)} [/tex]

Hope you could get an idea from here.

Doubt clarification - use comment section.

Two trucks, A and B, start from the intersection C of two straight roads at the same time. Truck A is traveling twice as fast as truck B, and after 4 hours, the two trucks are 350 miles apart. Find the approximate speed of truck B in miles per hour.


A. 39 B. 51 C. 44 D. 78

Answers

Answer:

6

Step-by-step explanation:

Other Questions
Cliek an item in the list or group of pictures at the bottom of the problem and, holding the button down, drag it into thecorrect position in the answer box. Release your mouse button when the item is place. If you change your mind, dragthe item to the trashcan click the trashcan to clear all your answersFactor completely, then place the answer in the proper location on the grid2107.57a+ 30123456789 Will mark brainly :)Which interval contains all the x-values where f(x)=log0.75x is positive?x>0.750 10 if EF = 8x + 16, FG = 60, and eg = 156, find the value of x how to tell if something is a strong or weak acid or base Out of 250 million cars on the road in US, about 3.5% are diesel. What percentage are not diesel? what is one reason many mexicans did not want to colonize texas? An elevator company claims "for every 15 inches of cable, we can hold up to 32 pounds". Each elevator comes with a standard length of 18 inches for their cable. If an elevator has aweight limit of 800 lbs, how many additional inches of cable were used? A bullet of mass 20 g has its kinetic energy equal to 400 J. Find the speed of the bullet. Katarina bought a package of 500 stickers. 25% of the stickers are hearts. The remaining stickers are stars. Select all the statements that are true. Someone please help meh List the angles of abc from smallest to largest. Ab = 20, bc = 12, ca = 10. Which inequality does the graph represent? What happens during photosynthesis?a. Autotrophs consume sugarsb. Autotrophs produce sugarsc. Heterotrophs produce ATP.d. Heterotrophs consume ATP. Which metals can be extracted from following ores (i) Argentite (ii) Haematite (iii) Chalcopyrite (iv) Bauxite Which direction did the ships sail to get from North America to Europe how does one determine a molecular formula from the epirical formula Answer my question for the ones who knows and speaks Spanish plzzz!!! what are some expressions that are equivalent to 3(7x6). noah does half as many push-ups as joseph. joshua does 3 times as many push-ups as joseph. they complete 324 altogether. How many push-ups does joshua complete? What theme in the novel does Melville allude to with his use of the word vindictive to describe the movement of the ship in this excerpt?inequalityrebirthignorancerevenge